Menu Close

Category: Others

A-motorcyclist-passing-a-road-junction-moves-due-east-for-8-seconds-at-a-uniform-speed-of-5-m-s-He-then-moves-due-north-for-another-6-seconds-with-the-same-speed-At-the-end-of-6-seconds-his-displ

Question Number 11968 by tawa last updated on 08/Apr/17 $$\mathrm{A}\:\mathrm{motorcyclist},\:\mathrm{passing}\:\mathrm{a}\:\mathrm{road}\:\mathrm{junction}\:,\:\mathrm{moves}\:\mathrm{due}\:\mathrm{east}\:\mathrm{for}\:\mathrm{8}\:\mathrm{seconds}\:\mathrm{at} \\ $$$$\mathrm{a}\:\mathrm{uniform}\:\mathrm{speed}\:\mathrm{of}\:\mathrm{5}\:\mathrm{m}/\mathrm{s}.\:\mathrm{He}\:\mathrm{then}\:\mathrm{moves}\:\mathrm{due}\:\mathrm{north}\:\mathrm{for}\:\mathrm{another}\:\mathrm{6}\:\mathrm{seconds} \\ $$$$\mathrm{with}\:\mathrm{the}\:\mathrm{same}\:\mathrm{speed}.\:\mathrm{At}\:\mathrm{the}\:\mathrm{end}\:\mathrm{of}\:\mathrm{6}\:\mathrm{seconds}\:\mathrm{his}\:\mathrm{displacement}\:\mathrm{from}\:\mathrm{the} \\ $$$$\mathrm{road}\:\mathrm{junction}\:\mathrm{is}\:\mathrm{50}\:\mathrm{m}\:\mathrm{in}\:\mathrm{the}\:\mathrm{diretion}\:\mathrm{of} \\ $$$$\left.\mathrm{A}\right)\:\mathrm{53}°\mathrm{E}\:\:\left(\mathrm{B}\right)\:\:\mathrm{37}°\mathrm{E}\:\:\left(\mathrm{C}\right)\:\:\mathrm{53}°\mathrm{W}\:\:\left(\mathrm{D}\right)\:\:\mathrm{37}°\mathrm{W} \\ $$ Terms of Service Privacy Policy…

A-load-of-60-kg-is-pushed-up-a-400-m-incline-side-of-a-platform-3-m-high-what-is-the-velocity-ratio-of-the-plane-

Question Number 11964 by tawa last updated on 07/Apr/17 $$\:\mathrm{A}\:\mathrm{load}\:\mathrm{of}\:\mathrm{60}\:\mathrm{kg}\:\mathrm{is}\:\mathrm{pushed}\:\mathrm{up}\:\mathrm{a}\:\mathrm{400}\:\mathrm{m}\:\mathrm{incline}\:\mathrm{side}\:\mathrm{of}\:\mathrm{a}\:\mathrm{platform}\:\mathrm{3}\:\mathrm{m}\:\mathrm{high}.\: \\ $$$$\mathrm{what}\:\mathrm{is}\:\mathrm{the}\:\mathrm{velocity}\:\mathrm{ratio}\:\mathrm{of}\:\mathrm{the}\:\mathrm{plane}\:? \\ $$ Terms of Service Privacy Policy Contact: info@tinkutara.com

Let-a-and-b-be-two-numbers-x-be-the-single-arithmetic-mean-of-a-and-b-Show-that-the-sum-of-n-arithmetic-means-between-a-and-b-is-nx-

Question Number 11930 by 786786AM last updated on 05/Apr/17 $$\mathrm{Let}\:\mathrm{a}\:\mathrm{and}\:\mathrm{b}\:\mathrm{be}\:\mathrm{two}\:\mathrm{numbers},\:\mathrm{x}\:\mathrm{be}\:\mathrm{the}\:\mathrm{single}\:\mathrm{arithmetic}\:\mathrm{mean}\:\mathrm{of}\:\mathrm{a}\:\mathrm{and}\:\mathrm{b}. \\ $$$$\mathrm{Show}\:\mathrm{that}\:\mathrm{the}\:\mathrm{sum}\:\mathrm{of}\:\mathrm{n}\:\mathrm{arithmetic}\:\mathrm{means}\:\mathrm{between}\:\mathrm{a}\:\mathrm{and}\:\mathrm{b}\:\mathrm{is}\:\mathrm{nx}. \\ $$ Answered by ajfour last updated on 05/Apr/17 $${A}_{{r}} =\:{a}+\frac{\left({b}−{a}\right)}{\left({n}+\mathrm{1}\right)}{r} \\ $$$$\underset{{r}=\mathrm{1}}…

a-You-are-listening-to-your-favourite-song-on-a-CD-You-note-that-the-sound-wave-has-a-pleasant-frequency-of-12-Hertz-i-What-is-the-velovity-of-the-sound-wave-ii-What-wavelenght-are-the-wave-mo

Question Number 11898 by tawa last updated on 04/Apr/17 $$\left(\mathrm{a}\right) \\ $$$$\mathrm{You}\:\mathrm{are}\:\mathrm{listening}\:\mathrm{to}\:\mathrm{your}\:\mathrm{favourite}\:\mathrm{song}\:\mathrm{on}\:\mathrm{a}\:\mathrm{CD}.\:\mathrm{You}\:\mathrm{note}\:\mathrm{that}\:\mathrm{the}\:\mathrm{sound} \\ $$$$\mathrm{wave}\:\mathrm{has}\:\mathrm{a}\:\mathrm{pleasant}\:\mathrm{frequency}\:\mathrm{of}\:\mathrm{12}\:\mathrm{Hertz}. \\ $$$$\left(\mathrm{i}\right)\:\mathrm{What}\:\mathrm{is}\:\mathrm{the}\:\mathrm{velovity}\:\mathrm{of}\:\mathrm{the}\:\mathrm{sound}\:\mathrm{wave} \\ $$$$\left(\mathrm{ii}\right)\:\mathrm{What}\:\mathrm{wavelenght}\:\mathrm{are}\:\mathrm{the}\:\mathrm{wave}\:\mathrm{moving}\:\mathrm{at} \\ $$$$\left(\mathrm{iii}\right)\:\mathrm{What}\:\mathrm{is}\:\mathrm{the}\:\mathrm{period} \\ $$$$\left(\mathrm{b}\right) \\ $$$$\mathrm{60}\:\mathrm{complete}\:\mathrm{waves}\:\mathrm{pass}\:\mathrm{a}\:\mathrm{particular}\:\mathrm{point}\:\mathrm{in}\:\mathrm{4}\:\mathrm{secs},\:\mathrm{if}\:\mathrm{the}\:\mathrm{distance}\:\mathrm{between} \\…

Draw-the-structural-formula-of-the-compound-2-2-7-trimethyl-4-1-methylpropyl-nonane-

Question Number 11883 by tawa last updated on 03/Apr/17 $$\mathrm{Draw}\:\mathrm{the}\:\mathrm{structural}\:\mathrm{formula}\:\mathrm{of}\:\mathrm{the}\:\mathrm{compound} \\ $$$$\mathrm{2},\mathrm{2},\mathrm{7}\:-\:\mathrm{trimethyl}\:-\:\mathrm{4}\:-\:\left(\mathrm{1}\:-\:\mathrm{methylpropyl}\right)\:\mathrm{nonane} \\ $$ Answered by sandy_suhendra last updated on 04/Apr/17 Commented by tawa last…

A-sample-of-steam-at-140-bar-is-states-to-have-enthalpy-of-3009-1-kJ-kg-Calculate-the-internal-energy-and-entropy-

Question Number 11828 by tawa last updated on 01/Apr/17 $$\mathrm{A}\:\:\mathrm{sample}\:\mathrm{of}\:\mathrm{steam}\:\mathrm{at}\:\mathrm{140}\:\mathrm{bar}\:\mathrm{is}\:\mathrm{states}\:\mathrm{to}\:\mathrm{have}\:\mathrm{enthalpy}\:\mathrm{of}\:\:\mathrm{3009}.\mathrm{1}\:\mathrm{kJ}/\mathrm{kg}, \\ $$$$\mathrm{Calculate}\:\mathrm{the}\:\mathrm{internal}\:\mathrm{energy}\:\mathrm{and}\:\mathrm{entropy}. \\ $$ Terms of Service Privacy Policy Contact: info@tinkutara.com

What-is-the-velocity-of-sound-at-100-C-if-the-velocity-of-sound-at-0-C-is-340-m-s-

Question Number 11791 by tawa last updated on 31/Mar/17 $$\mathrm{What}\:\mathrm{is}\:\mathrm{the}\:\mathrm{velocity}\:\mathrm{of}\:\mathrm{sound}\:\mathrm{at}\:\mathrm{100}°\mathrm{C}\:\:\mathrm{if}\:\mathrm{the}\:\mathrm{velocity}\:\mathrm{of}\:\mathrm{sound}\:\mathrm{at}\:\mathrm{0}°\mathrm{C}\:\mathrm{is}\:\mathrm{340}\:\mathrm{m}/\mathrm{s} \\ $$ Answered by mrW1 last updated on 31/Mar/17 $${v}_{{t}} ={v}_{\mathrm{0}} \sqrt{\mathrm{1}+\frac{{t}}{\mathrm{273}}} \\ $$$$=\mathrm{340}\sqrt{\mathrm{1}+\frac{\mathrm{100}}{\mathrm{273}}} \\…

given-3-y-1-6-2-x-3-y-x-2-find-1-x-1-y-

Question Number 77186 by jagoll last updated on 04/Jan/20 $$\mathrm{given}\: \\ $$$$\begin{cases}{\mathrm{3}^{\mathrm{y}} −\mathrm{1}=\:\frac{\mathrm{6}}{\mathrm{2}^{\mathrm{x}} }}\\{\left(\mathrm{3}\right)^{\frac{\mathrm{y}}{\mathrm{x}}} \:=\:\mathrm{2}\:}\end{cases}\:\:\mathrm{find}\:\frac{\mathrm{1}}{\mathrm{x}}+\frac{\mathrm{1}}{\mathrm{y}}. \\ $$ Answered by john santu last updated on 04/Jan/20…